You are on page 1of 67

Section Name : Analytical and Numerical Ability

Type: MCQ Single    |    Marks: +1/-0   |    Topic: Quant > Percentages    |    Time


Spent: 
1 Mins 14 Secs
1. 
In a class, 55% of the students play cricket, 60% play volley ball and 70% play foot ball.
Each student plays at least one of the three games.
What is the maximum possible percentage of students who play all the three games?
A. 42
B. 43
C. 85
D. 55

Status :  

Incorrect
Given Answer : C
Answer : A
View Solution
Let 'a' represent the people who play exactly one game, 'b' represent the people who
play exactly two games and 'C' repesents, the people who play exactly three games.
Thus,
a + b + c = 100 ....(1)
and a + 2b + 3c = 185 ....(2)
From (1) and (2)
b + 2c = 85
to maximize 'C' we have to minimize 'b'
the minimum value, for b is 1, as 2C must be an integer.
Thus, C = 42
Bookmark Share question feedback

Type: MCQ Single    |    Marks: +1/-0   |    Topic: Quant > Geometry    |    Time Spent: 


1 Mins 14 Secs
2. 

In the given figure, ABC is a triangle. BCP is a straight line. What is the value of x?

A. 30o
B. 45o
C. 40o
D. 35o

Status :  

Correct
Answer : C
View Solution
In any triangle exterior angle = sum of the interior opposite angles,
x + 2x = 120
x = 40o
Bookmark Share question feedback

Type: MCQ Single    |    Marks: +1/-0   |    Topic: Quant > Average    |    Time Spent: 


0 Mins 48 Secs
3. 
The average age of students of a class and their class teacher, who is 44 years old, is
22 years. Which of the following can be the average age (in years) of the students?
A. 22.5
B. 23
C. 21
D. 20.5

Status :  

Incorrect
Given Answer : D
Answer : C
View Solution
Let the number of the students in the class be N.
Let the average age of the students be A years.

Average age of the to class of the students and their class teacher = \frac { AN+44 }{
N+1 } =22N+1AN+44=22
AN+44=22N+22
AN=22 (N-1)
N > N-1
Therefore AN=22(N-1)
=> A > 22
Therefore among the choices, A can be 21 or 20.5
But N must be an integer.

A=21 results in N=22 which A=20.5 results in N = \frac { 44 }{ 3 }344 which is not


possible
Therefore A = 21
Bookmark Share question feedback

Type: MCQ Single    |    Marks: +1/-0   |    Topic: Quant > Average    |    Time Spent: 


1 Mins 6 Secs
4. 
A get-together was attended by 40 gentlemen and 70 ladies. The average height of the
gentlemen was 183 cm. The average height of all the persons was 169 cm. Find the
average height (in cm) of the ladies.
A. 161
B. 163
C. 159
D. 157

Status :  

Correct
Answer : A
View Solution
Let the average height of the ladies be \ellℓ cm.
Average height of all the persons

= \frac { 183.40+\ell 70 }{ 40+70 } cm=169cm40+70183.40+ℓ70cm=169cm


183.40+70l = 169 (110)
7320 =70l = 18590
70l = 11270
l = 161
Bookmark Share question feedback

Type: MCQ Single    |    Marks: +1/-0   |    Topic: Quant > Mixtures And Alligations    |  


Time Spent: 
2 Mins 42 Secs
5. 
A vessel has 90 litres of pure milk. 9 litres of its contents was withdrawn and replaced
with water. This process was repeated once. Find the quantity (in litres) of milk in the
vessel at the end of this repetition.
A. 72.9
B. 69
C. 70.2
D. 72

Status :  

Unanswered
Answer : A
View Solution
Quantity of milk in the vessel just after 9 liters of pure milk was withdrawn = (90-9)
liters = 81 liters just after water was then added, the vessel would have had 81 liters of
milk in 90 liters of mixtures.
Therefore 9/10 the of its contents would have been milk been and 9 liters of the mixture
was then withdrawn.
Therefore just after the withdrawn 9/10(81) = 72.9 liters of milk remained in the vessel.
Then water was added, this quantity of milk would be there in 90 liters of the mixture.
Therefore required quantity = 72.9 liters
Bookmark Share question feedback
Type: MCQ Single    |    Marks: +1/-0   |    Topic: Quant > Trigonometry    |    Time
Spent: 
0 Mins 41 Secs
6. 
From the top of a building 30m high, the angle of elevation of the top of a tower is found
to be equal to the angle of depression of the foot of the tower. Find the height of the
tower.
A. 15 m
B. 30 m
C. 60 m
D. 90 m

Status :  

Unanswered
Answer : C
View Solution

Let AB=h and CD = x

From \triangle CDE△CDE
tan\theta =\frac { DE }{ CD }tanθ=CDDE
tan\theta =\frac { 30 }{ x }tanθ=x30
x = \frac { 30 }{ tan\theta }tanθ30 =BE
From \triangle ABE△ABE

tan\theta =\frac { AB }{ BE }tanθ=BEAB

tan\theta =\frac { h }{ 30/tan\theta } \Rightarrow h=tan\theta \times


30/tan\thetatanθ=30/tanθh⇒h=tanθ×30/tanθ

h=30m

Therefore height of the tower is AB + BC = 30 + 30 = 60 m


Bookmark Share question feedback
Type: MCQ Single    |    Marks: +1/-0   |    Topic: Quant > Geometry    |    Time Spent: 
1 Mins 58 Secs
7. 
The perimeter of the triangle whose vertices are (3, 2), (7, 2) and (7, 5) (in units) is
A. 5
B. 7
C. 9
D. 12

Status :  

Unanswered
Answer : D
View Solution
Perimeter = Sum of distance between the adjacent points

\sqrt { { 4 }^{ 2 }+{ 0 }^{ 2 }+ } \sqrt { { 0 }^{ 2 }+3^{ 2 }+ } \sqrt { { 4 }


^{ 2 }+{ 3 }^{ 2 } } =4+3+5=1242+02+02+32+42+32=4+3+5=12
Bookmark Share question feedback

Type: MCQ Single    |    Marks: +1/-0   |    Topic: Quant > Equations    |    Time Spent: 


0 Mins 29 Secs
8. 
If the roots of 2x2 + 5x + 1 = 0 are each increased by 2, then the equation with these
values as roots is
A. 2x2+ 3x + 1 = 0
B. 2x2+ 3x - 1 = 0
C. 2x2- 3x + 1 = 0
D. 2x2- 3x - 1 = 0

Status :  

Incorrect
Given Answer : C
Answer : D
View Solution
Required equation is obtained by replacing x by x -2
i.e., 2(x-2)2+5(x-2)+1=0
i.e., 2x2 - 3x-1=0
Bookmark Share question feedback

Type: MCQ Single    |    Marks: +1/-0   |    Topic: Quant > Statistics    |    Time Spent: 


1 Mins 2 Secs
9. 
If the difference between the mode and the median is 16, then the difference between
the mode and the mean is
A. 20
B. 32
C. 33
D. 
24
Status :  

Incorrect
Given Answer : B
Answer : D
View Solution
\frac { Mode-Median }{ Mode-Mean } =\frac { 2 }{ 3 }Mode−MeanMode−Median=32
\frac { 16 }{ x } =\frac { 2 }{ 3 } \Rightarrow x=\frac { 3 }{ 2 } \times
16=24x16=32⇒x=23×16=24
Bookmark Share question feedback

Type: MCQ Single    |    Marks: +1/-0   |    Topic: Quant > Set Theory    |    Time


Spent: 
2 Mins 59 Secs
10. 
If P(A)=0.32, P(B)=0.43 and P(A\cup B)=0.58,thenP(\overline { A } \cap B)
(A∪B)=0.58,thenP(A∩B) =
A. 0.15
B. 0.11
C. 0.26
D. 0.17

Status :  

Incorrect
Given Answer : D
Answer : C
View Solution
P(A\cap B)=P(A)+P(B)-P(A\cup B)(A∩B)=P(A)+P(B)−P(A∪B) = 0.32 + 0.43 -
0.58 = 0.17

P(\overline { A } \cap B)=P(B)-P(A\cap B)(A∩B)=P(B)−P(A∩B) =0.43 - 0.17 =


0.26
Bookmark Share question feedback

Type: MCQ Single    |    Marks: +1/-0   |    Topic: Quant > Percentages    |    Time


Spent: 
1 Mins 9 Secs
11. 
The salary of A is 25% more than the salary of B. By what percentage is the salary of B
less than the salary of A?
A. 20%
B. 25%
C. 30%
D. 40%

Status :  

Correct
Answer : A
View Solution
Let the salary of B be Rs.100. As the salary of A is 25% more than that of B it will be
Rs.125.

The salary of B is less than the salary of A by \frac { 25 }{ 125 }12525 x 100 i.e., 20%.
Bookmark Share question feedback

Type: MCQ Single    |    Marks: +1/-0   |    Topic: Quant > Percentages    |    Time


Spent: 
2 Mins 6 Secs
12. 
Ajay and Vijay have some marbles with them. If 10% of the marbles with Ajay equals
30% of the marbles with Vijay and they have a total of 80 marbles, the number of
marbles with Vijay is
A. 10
B. 15
C. 20
D. 25

Status :  

Unanswered
Answer : C
View Solution
Let the number of marbles with Ajay and Vijay be A and V respectively.

Given, \frac { 10 }{ 100 } A10010A = \frac { 30 }{ 100 } V10030V


=> A = 3V
Also, A + V = 80
=> 3V + V = 80
V = 20.
Bookmark Share question feedback

Type: MCQ Single    |    Marks: +1/-0   |    Topic: Quant > Ratio And Proportion    |  


Time Spent: 
0 Mins 55 Secs
13. 
In a class the total number of boys and girls is 81. Which of the following can't be the
ratio of the number of boys and girls?
A. 1 : 2
B. 5 : 3
C. 7 : 2
D. 20 : 7

Status :  

Incorrect
Given Answer : D
Answer : B
View Solution
Going by the choices, the sum of the terms of the ratio in choice (2) is 8. But 81 is not,
divisible by 8. So 5 : 3 is not possible.
Bookmark Share question feedback

Type: MCQ Single    |    Marks: +1/-0   |    Topic: Quant > Ratio And Proportion    |  


Time Spent: 
2 Mins 4 Secs
14. 
The monthly expenses of A and B are in the ratio 4 : 3. Their monthly savings are in the
ratio 3 : 2. Find the combined monthly savings of A and B, given, their incomes
are र 10000 and र 7000 respectively.
A. र 60,000
B. र 40,000
C. र 10,000
D. र 5,000

Status :  

Correct
Answer : C
View Solution

Bookmark Share question feedback

Type: MCQ Single    |    Marks: +1/-0   |    Topic: Quant > Mixtures And Alligations    |  


Time Spent: 
0 Mins 51 Secs
15. 
In what ratio two milk solutions having 80% pure milk and 50% pure milk should be
mixed such that the resultant solution will have 68% pure milk?
A. 2 : 3
B. 3 : 2
C. 5 : 6
D. 6 : 5

Status :  

Correct
Answer : B
View Solution
Bookmark Share question feedback

Type: MCQ Single    |    Marks: +1/-0   |    Topic: Quant > Equations    |    Time Spent: 


1 Mins 3 Secs
16. 
What is the number of solutions for the equations 2x + 3y = 25 and 6x + 9y = 75?
A. 0
B. 1
C. 2
D. infinite

Status :  

Incorrect
Given Answer : A
Answer : D
View Solution
Since the ratio of the coefficients of x in the first and the second equations is 1:3
and the ratio of the coefficients of y in the first and the second equation is also 1:3 the'
set of equation will have infinite will have infinite solutions.
Bookmark Share question feedback

Passage :
Type: MCQ Single    |    Marks: +1/-0   |    Topic: Data Interpretation > Bar Graphs    |  
Time Spent: 
2 Mins 43 Secs
17. 
Find the average annual growth rate in the exports over the given period approximately
to two decimal places.
A. 3.42%
B. 2.38%
C. 1.95%
D. 2.96%

Status :  

Unanswered
Answer : B
View Solution

Bookmark Share question feedback

Passage :
Type: MCQ Single    |    Marks: +1/-0   |    Topic: Data Interpretation > Bar Graphs    |  
Time Spent: 
0 Mins 57 Secs
18. 
During which year was the ratio between the imports and the exports, the least?
A. 2000
B. 2003
C. 2005
D. 2006

Status :  

Incorrect
Given Answer : A
Answer : D
View Solution
From the given graph, it can be seen that imports is less than exports in 2000, 2003,
2005 and 2006
Among these years, 2006 has the least imports and the greatest exports.
Least ratio will occur in 2006.
Bookmark Share question feedback

Passage :
Type: MCQ Single    |    Marks: +1/-0   |    Topic: Data Interpretation > Tables    |  
Time Spent: 
1 Mins 16 Secs
19. 
Find the total number of females who died in accidents in the five districts together in
2008.
A. 3673
B. 3579
C. 3484
D. 3797

Status :  

Correct
Answer : B
View Solution
The required total = 1004 + 1008 + 627 + 417 + 523
= 3579.
Bookmark Share question feedback

Passage :
Type: MCQ Single    |    Marks: +1/-0   |    Topic: Data Interpretation > Tables    |  
Time Spent: 
2 Mins 20 Secs
20. 
The number of males who died due to accidents in district A in 2003 is approximately
what percent of the total number of males who died due to accidents in district A over
the given period?
A. 14%
B. 15%
C. 11%
D. 13%

Status :  

Incorrect
Given Answer : B
Answer : D
View Solution
The total number of males died in district A = 68 + 856 + 772 + 961 + 963 + 1168
= 5400
The required percentage =

\frac { 680 }{ 5400 } (100)5400680(100) % = \frac { 680 }{ 54 }54680 % = 13%


Bookmark Share question feedback

Passage :

Type: MCQ Single    |    Marks: +1/-0   |    Topic: Data Interpretation > Line Graphs    |  


Time Spent: 
0 Mins 36 Secs
21. 
In the year 2010, what percentage of the total number of employees in the public sector
were employed in the state government?
A. 21.37%
B. 29.43%
C. 42.37%
D. 37.31%

Status :  

Unanswered
Answer : D
View Solution
In the years 2010, the total number of employees
= 3550 + 7275 + 6425 + 2250 = 19500

The required percentage = \frac { 7275 }{ 19500 }195007275 x 100


= 37.31%
Bookmark Share question feedback

Passage :

Type: MCQ Single    |    Marks: +1/-0   |    Topic: Data Interpretation > Line Graphs    |  


Time Spent: 
2 Mins 41 Secs
22. 
What is the ratio of the total number of central and quasi government employees in 2009
to those working in local bodies and state government in 2011?
A. 191 : 195
B. 191 : 197
C. 181 : 195
D. 171 : 185
Status :  

Correct
Answer : A
View Solution
The required ratio
= (3350 + 6200) : (2300 + 7450)
= 9550 : 9750 = 191 : 195
Bookmark Share question feedback

Passage :

Type: MCQ Single    |    Marks: +1/-0   |    Topic: Data Interpretation > Line Graphs    |  


Time Spent: 
0 Mins 39 Secs
23. 
During the year 2008, the number of employees of which of the following organisations
is closest to the average number of employees per organisation in that year?
A. Central government
B. State government
C. Quasi government
D. Local bodies

Status :  

Incorrect
Given Answer : D
Answer : C
View Solution
average number of employees per organisation in 2008

= \frac { 3200+6450+4750+2200 }{ 4 }43200+6450+4750+2200 = 4150


4750 is the nearest value 4150.
Bookmark Share question feedback

Type: MCQ Single    |    Marks: +1/-0   |    Topic: Logical Reasoning > Directions    |  


Time Spent: 
1 Mins 40 Secs
24. 
Starting from O, Vivek walked 40 m towards south, then he turned left and walked 60 m.
He again turned left and walked 40 m. He once again turned left and walked 80 m and
reached at D. How far and in which direction is D from O?
A. 20 m, east
B.  40 m, west
C. 20 m, west
D. 20 m, south

Status :  

Correct
Answer : C
View Solution

Bookmark Share question feedback

Type: MCQ Single    |    Marks: +1/-0   |    Topic: Logical Reasoning > Clocks    |  


Time Spent: 
0 Mins 16 Secs
25. 
A clock strikes once at 1O' clock, twice at 2O'clock, thrice at 3O' clock and so on. At 5O'
clock if it takes 8 seconds to strike five times then, how long (in seconds) does it take to
strike eleven times at 11'O clock?
A. 22
B. 18
C. 20
D. 24

Status :  

Unanswered
Answer : C
View Solution
Bookmark Share question feedback

Type: MCQ Single    |    Marks: +1/-0   |    Topic: Logical Reasoning > Syllogism    |  


Time Spent: 
1 Mins 0 Secs
26. 
The following question consists of three statements followed by two conclusions.
Consider the statements to be true even if they vary from the commonly known facts and
find out which of the conclusions logically follow/s the given statements and choose the
proper alternative from the given choices.

Statements:
All pens are pencils.
All pens are erasers.
All erasers are chalks.

Conclusions:
I. All erasers are pens.
II. Some erasers are pens.
A. If only I follows.
B. If only II follows.
C. If neither I nor II follows.
D. If both I and II follows.

Status :  

Incorrect
Given Answer : C
Answer : B
View Solution

Bookmark Share question feedback


Passage :
Directions for question: Select the correct alternative from the given choices.
Type: MCQ Single    |    Marks: +1/-0   |    Topic: Verbal > Odd Sentence Out    |    Time
Spent: 
0 Mins 29 Secs
27. 
Pick the odd one out.
A. 145
B. 225
C. 197
D. 257

Status :  

Unanswered
Answer : B
View Solution

Bookmark Share question feedback

Passage :
Directions for question: Select the correct alternative from the given choices.
Type: MCQ Single    |    Marks: +1/-0   |    Topic: Logical Reasoning > Calendars    |  
Time Spent: 
0 Mins 35 Secs
28. 
For a car to Manufacture, it has to go through 3 steps in the following order.
Step I: Designing, takes 19 days.
Step II: Manufacturing, takes 24 days.
Step III: Testing, takes 6 days.
Only after completion of one step, the next step starts in the following day. If step II
started on Friday, then on which day did step I start and step III end?
A. Sunday, Saturday
B. Thursday, Saturday
C. Saturday, Sunday
D. Saturday, Thursday

Status :  

Unanswered
Answer : A
View Solution

Bookmark Share question feedback


Type: MCQ Single    |    Marks: +1/-0   |    Topic: Logical Reasoning > Series    |    Time
Spent: 
0 Mins 20 Secs
29. 
The value of the 12th term in the serves -1, -1-2, -1 -2 -3, -1 - 2 -3 -4, _______.
A. -64
B. 96
C. -84
D. -78

Status :  

Incorrect
Given Answer : C
Answer : D
View Solution

Bookmark Share question feedback

Type: MCQ Single    |    Marks: +1/-0   |    Topic: Logical Reasoning > Series    |    Time


Spent: 
1 Mins 6 Secs
30. 
Find the next term in the series.
RGHI, VKLM, ZOPQ, ______
A. BRSY
B. CSTV
C. DSRZ
D. DSTU

Status :  

Correct
Answer : D
View Solution

Bookmark Share question feedback


Section Name : Verbal Ability And Reading Comprehension
Passage :
Directions for question: Identify the correct sentence or sentences:
Type: MCQ Single    |    Marks: +1/-0   |    Topic: Verbal > Sentence Correction    |  
Time Spent: 
0 Mins 56 Secs
1. 
(a) Organizational cultures take root in a myriad ways.
(b) Some are shaped by a forceful leader.
(c) Others take shape influenced by an industry ethic or a place.
(d) Yet others take shape as a response to its environment.
A. Only (a)
B. Only (c)
C. (b) and (c)
D. (a) and (d)

Status :  

Correct
Answer : C
View Solution
Sentence a must be either 'in myriad ways' or 'in a myriad of ways'. Sentence d must
have 'their' (plural) not 'its' (singular) since the reference is to 'other'. b and c are' correct.
Bookmark Share question feedback

Passage :
Directions for question: The given sentence has four underlined parts. One of them
has a mistake. Mark the letter of the wrong part as the answer.
Type: MCQ Single    |    Marks: +1/-0   |    Topic: Verbal > Sentence Correction    |  
Time Spent: 
1 Mins 4 Secs
2. 
Young men and women (A) do not seem (B) to have a desire in sustaining (C) their
marriage. (D)
A. (A)
B. (B)
C. (C)
D. (D)

Status :  

Incorrect
Given Answer : B
Answer : C
View Solution
The word 'desire' is followed by to + infinitive. Hence correction should be' .... seem to,
have a desire to sustain their marriage'.
Bookmark Share question feedback

Type: MCQ Single    |    Marks: +1/-0   |    Topic: Verbal > Idioms And Phrases    |  


Time Spent: 
0 Mins 45 Secs
3. 
Directions for question: Every option has an idiom which is explained correctly in one
of the four choices that follow. Pick out the correct choice.
A. 
to have a brush with the law - To encounter, or nearly encounter
B. 
in the bargain - to do bargaining
C. 
Fall by the wayside - getting into something
D. 
Burden of proof - to prove something

Status :  

Incorrect
Given Answer : B
Answer : A
View Solution
have a brush with (something)
To encounter, or nearly encounter, someone or something.
I had a brush with death when I was in that car accident.
Oh, you'll have a brush with the law sooner or later if you keep robbing houses.
Option A is the correct answer.
 
Bookmark Share question feedback

Passage :
Directions for question: Pick the most effective word from the given words to fill in the
blanks to make the sentence meaningfully complete.
Type: MCQ Single    |    Marks: +1/-0   |    Topic: Verbal > Fill In The Blanks    |    Time
Spent: 
0 Mins 39 Secs
4. 
In ______ of her talents, many awards have come her way, but it is her ability to ______
a passion that she finds more rewarding.
A. honour, produce
B. view, create
C. recognition, pursue
D. anticipation, organize

Status :  

Correct
Answer : C
View Solution
She was given many awards. This was done in 'recognition' of her talents, not in
'anticipation' of her talents. It can be to 'honour' her talents but not just in 'view' of,
talents. But she finds 'pursuing' a passion to be more rewarding. Hence option (C).
Bookmark Share question feedback
Passage :
Directions for question: The question gives a word followed by four choices. From the
choices, select the most suitable synonym (word which means the same) for the main
word.
Type: MCQ Single    |    Marks: +1/-0   |    Topic: Verbal > Synonyms    |    Time
Spent: 
0 Mins 17 Secs
5. 
GROTESQUE
A. bulky
B. murky
C. cumbersome
D. weird

Status :  

Correct
Answer : D
View Solution
'Grotesque' means weird.
Bookmark Share question feedback

Passage :
Directions for question: The question has a word followed by four choices. From the
choices, identify the one which is opposite in meaning (antonym) to the main word.
Type: MCQ Single    |    Marks: +1/-0   |    Topic: Verbal > Antonyms    |    Time Spent: 
0 Mins 10 Secs
6. 
FORTITUDE
A. chivalry
B. eruption
C. cowardice
D. chutzpah

Status :  

Correct
Answer : C
View Solution
'Fortitude' means courage. Hence its antonym is 'cowardice'.
Bookmark Share question feedback

Passage :
Directions for question: Read the following sentence and choose the best replacement
for the underlined part of the sentence from among the options given.
Type: MCQ Single    |    Marks: +1/-0   |    Topic: Verbal > Sentence Improvement    |  
Time Spent: 
0 Mins 27 Secs
7. 
The minister hit up at the persons who tried to malign his reputation.
A. hit on
B. hit back
C. hit around
D. hit upon

Status :  

Correct
Answer : B
View Solution
'Hit back' is the most appropriate phrasal verb to be used here. To hit back is to attack,
or criticize someone who has attacked or criticized you.
Bookmark Share question feedback

Passage :
Directions for question: Match the idioms given in column A with correct meanings in
column B.
Type: MCQ Single    |    Marks: +1/-0   |    Topic: Verbal > Idioms And Phrases    |  
Time Spent: 
0 Mins 49 Secs
8. 

A. 1 - (a), 2 - (c), 3 - (d), 4 - (b)


B. 1 - (b), 2 - (c), 3 - (a), 4 - (d)
C. 1 - (a), 2 - (d), 3 - (b), 4 - (c)
D. 1 - (b), 2 - (a), 3 - (d), 4 - (c)

Status :  

Incorrect
Given Answer : A
Answer : D
View Solution
Only (D) has the correct pairs.
Bookmark Share question feedback

Passage :
Directions for question: Fill in the blank with the appropriate word.
Type: MCQ Single    |    Marks: +1/-0   |    Topic: Verbal > Fill In The Blanks    |    Time
Spent: 
0 Mins 42 Secs
9. 
The judicial ______ ordered by the State Government into the incidents should cover all
aspects of the violence, including the complaints against some police personnel.
A. study
B. inquest
C. analysis
D. probe

Status :  

Incorrect
Given Answer : A
Answer : D
View Solution
The judicial probe ( an investigation) ordered by the state government should cover all
the aspects of violence and the complaints against some police personnel. The
remaining choices are not suitable to the context. Study: Investigation and analysis of a
subject
analysis: a detailed examination.
inquest: a judicial inquiry.
In the context it is all investigation made by the police. Hence choice (D) is the
appropriate answer.
Bookmark Share question feedback

Passage :
Directions for question: Select the correct alternative from the given choices.
Type: MCQ Single    |    Marks: +1/-0   |    Topic: Logical Reasoning > Statement
Conclusion    |    Time Spent: 
1 Mins 18 Secs
10. 
A company reported a significant rise in its sales subsequent to changing the telecast
time of its commercials from nonprime time to prime time. Hence it is concluded that for
continued increase in sales the company must continue to arrange for telecasting the
commercials during prime time even though it is costlier to do so.

Which of the following, if true, would most weaken the above conclusion?
A. The difference between the costs of telecasting the commercials during prime and
non prime time is only marginally less than the increased revenue due to increased
sales.
B. The company faced labour unrest for longer period during the time when the
telecasting timings of the commercials were changed.
C. The company had resorted to a different type of costing after it changed the telecast
timings of the commercials.
D. The chief competitor of the company closed down soon after timings were changed
for the telecast of the commercials.

Status :  

Incorrect
Given Answer : A
Answer : D
View Solution
Change of telecast timing of commercials is given as the reasons for the increased
sales. As (D) gives another possible explanation for the increase in sales. (D) is the
correct answer. (A) is incorrect as the revenue is not the issue. (B) deals with labour
unrest hence supply so is irrelevant to sales. As cost is not discussed (C) is wrong
similarly the cost telecasting is also an irrelevant issue.
Bookmark Share question feedback

Passage :
Directions for question: Fill in the blank with the appropriate word.
Type: MCQ Single    |    Marks: +1/-0   |    Topic: Verbal > Fill In The Blanks    |    Time
Spent: 
0 Mins 32 Secs
11. 
The old city area in Hyderabad is known to be prone to communal violence, but, despite
the early warnings and slow beginnings of the trouble, the police were found wanting in
their response to a ______ situation,
A. volatile
B. mercurial
C. capricious
D. erratic

Status :  

Correct
Answer : A
View Solution
The old city area in Hyderabad is known to be prone to communal violence but the
police were found lacking in their response to a volatile situation. (i.e., highly
unpredictable situation which is liable to change especially for the worse). The remaining
choices are not appropriate
mercurial: change in mood of a person.
capricious: unaccountable changes of mood or behavior.
erratic: irregular in pattern.
Bookmark Share question feedback

Passage :
Directions for question: Read the following sentence and from among the options
choose the best replacement for the underlined part of the sentence:
Type: MCQ Single    |    Marks: +1/-0   |    Topic: Verbal > Sentence Improvement    |  
Time Spent: 
0 Mins 31 Secs
12. 
The businessman was so optimistic that his losses would be recouped that he did not let
them cast him away.
A. cast him up
B. cast him about
C. cast him down
D. cast him in

Status :  

Correct
Answer : C
View Solution
Option(C) is the right choice. Cast means to feel depressed. Cast away means to be
stranded after a shipwreck, cast about means to search far and wide They do not fit
here., cast in does not exist.
Bookmark Share question feedback

Passage :
Directions for question: Read the following sentence and from among the options
select the best replacement for the underlined part of the sentence:
Type: MCQ Single    |    Marks: +1/-0   |    Topic: Verbal > Sentence Correction    |  
Time Spent: 
0 Mins 49 Secs
13. 
When the little boy lost consciousness after a fall and was admitted to hospital, the
doctor said he would come along with in two days.
A. come up
B. come out
C. come back
D. come round

Status :  

Incorrect
Given Answer : B
Answer : D
View Solution
Option (D) is the right choice come round is to regain consciousness. Come along is to
progress, to appear come up is to arise come out is to be known to reveal and come
back means to return. Only (D) fits the context.
Bookmark Share question feedback

Passage :
Directions for question: The following sentence has four parts. Identify the part that
has a grammatical error and mark that letter as your answer.
Type: MCQ Single    |    Marks: +1/-0   |    Topic: Verbal > Sentence Correction    |  
Time Spent: 
0 Mins 43 Secs
14. 
Psychological theories treat magic (A) / as a personal phenomena intended to meet (B) /
individual needs, as opposed to (C) / social needs serving a collective purpose (D).
A. A
B. B
C. C
D. D

Status :  

Incorrect
Given Answer : A
Answer : B
View Solution
Option (B) is erroneous. Phenomenon is singular and agrees with 'a'. Phenomena is the
plural form.
Bookmark Share question feedback

Passage :
Directions for question: Fill in the blank with the appropriate word.
Type: MCQ Single    |    Marks: +1/-0   |    Topic: Verbal > Fill In The Blanks    |    Time
Spent: 
0 Mins 40 Secs
15. 
A trivial dispute over a soiled currency note took a ______ proportion.
A. menacing
B. warning
C. mitigating
D. assuaging

Status :  

Incorrect
Given Answer : B
Answer : A
View Solution
Menacing is the most appropriate word it means (threating, frightening, intimidating)' i.e.,
according to the context we can understand that a trivial dispute has led to frightening
consequences. Hence choice (a) the remaining words are not suitable to the context.
warning : a caution
mitigating : make less severe
assuaging : make less intense.
Bookmark Share question feedback

Passage :
Directions for question: Fill in the blank with the appropriate word.
Type: MCQ Single    |    Marks: +1/-0   |    Topic: Verbal > Fill In The Blanks    |    Time
Spent: 
0 Mins 31 Secs
16. 
That it ended in murder and arson is indicative of the deep ______ divide in Hyderabad.
A. religious
B. communal
C. sectorial
D. comical

Status :  

Incorrect
Given Answer : A
Answer : B
View Solution
The trivial issue has ended in a murder and it is indicative of the communal (conflict
between different communities, or different religions of ethic origins) divide in
Hyderabad. The remaining choices are not suitable to the context. sectorials Oflike a,
sector
religious : concerned with or believing a religion.
comical : amusing.
Bookmark Share question feedback

Passage :
Directions for question: Fill in the blank with the appropriate word.
Type: MCQ Single    |    Marks: +1/-0   |    Topic: Verbal > Fill In The Blanks    |    Time
Spent: 
0 Mins 15 Secs
17. 
Hyderabad has a long history of politics based on religious ______.
A. mechanisation
B. discrimination
C. polarisation
D. dramatisation

Status :  

Incorrect
Given Answer : B
Answer : C
View Solution
Hyderabad has a long history of politics based on religious polarisation (divide into two
sharply contrasting groups). The remaining choices are not suitable to the context
mechanisation : automatic devices.
discrimination : recognition and understanding the difference between two things.
dramatisation : exaggeration.
Bookmark Share question feedback

Passage :
Directions for question: Read the sentence to find out whether there is any error in it.
The error if any will be in one part of the sentence. The number of that part is the
answer.
Type: MCQ Single    |    Marks: +1/-0   |    Topic: Verbal > Sentence Correction    |  
Time Spent: 
0 Mins 38 Secs
18. 
The Indian IT industry (A) / is (B) / coping up (C) / with a lot of problems these days. (D)
A. A
B. B
C. C
D. D

Status :  

Incorrect
Given Answer : D
Answer : C
View Solution
The error is in part (C). The use of preposition 'up' with cope is incorrect.
Bookmark Share question feedback
Type: MCQ Single    |    Marks: +1/-0   |    Topic: Verbal > Fill In The Blanks    |    Time
Spent: 
0 Mins 23 Secs
19. 
Directions for question: Pick the most effective word from the given words to fill in the
blanks to make the sentence meaningfully complete.
Apart from having good _______ skills, the speaker had a _______ voice, which made
the people listen to him with rapt attention.
A. oratory, sonorous
B. speaking, gruff
C. lecturing, squeaky
D. preaching, deep

Status :  

Incorrect
Given Answer : B
Answer : A
View Solution
The person mentioned is a 'speaker' or an 'orator'. Hence it is understandable that he
will have 'oratory' skills. Also people will listen with rapt attention if the lecture is good
and if the voice of the speaker is 'sonorous' i.e. resonant and impressive. A 'gruff','
'squeaky' or 'deep' voice will not make people listen with rapt attention and hence
options (B), (C) and (D) can be ruled out.
Bookmark Share question feedback

Type: MCQ Single    |    Marks: +1/-0   |    Topic: Verbal > Fill In The Blanks    |    Time


Spent: 
0 Mins 28 Secs
20. 
Directions for question: Pick the most effective word from the given words to fill in the
blanks to make the sentence meaningfully complete.

A man who has _______ worldly life is not easily motivated to laugh because he has
willingly _______ himself from society.
A. sacrificed, ostracized
B. abandoned, exiled
C. renounced, isolated
D. forsaken, banished

Status :  

Correct
Answer : C
View Solution
A person can willingly 'renounce', 'abandon', 'forsake' or 'sacrifice' worldly life, Any of the
four words can be used for the fist blank, But a person cannot willingly 'banish' or 'exile'
himself from society, He doesn't 'ostracize' or exclude himself from society, He, 'isolates'
himself from society and that is why he is not easily motivated to behave like others do,
Hence the most appropriate pair of words would be 'renounced' and "isolated',
Bookmark Share question feedback
Passage :
Ancient Egypt developed a large, varied and fruitful medical tradition. Herodotus
described the Egyptians as "the healthiest of all men, next to the Libyans", because of
the dry climate and the notable public health system that they possessed. According to
him, "the practice of medicine is so specialized among them that each physician is a
healer of one disease and no more." Although Egyptian medicine, to a good extent, dealt
with the supernatural, it eventually developed a practical use in the fields of anatomy,
public health, and clinical diagnostics.
The Atharvaveda, a sacred text of Hinduism dating from the Early Iron Age, is one of the
first Indian text dealing with medicine, like the medicine of the Ancient Near East based
on concepts of the exorcism of demons and magic. The Atharvaveda also contain
prescriptions of herbs for various ailments. The use of herbs to treat ailments would later
form a large part of Ayurveda.
Ayurveda, meaning the "complete knowledge for long life" is another medical system of
India. Its two most famous texts belong to the schools of Charaka and Sushruta. The
earliest foundations of Ayurveda were built on a synthesis of traditional herbal practices
together with a massive addition of theoretical conceptualizations, new nosologies and
new therapies dating from about 600 BCE onwards, and coming out of the communities
of thinkers who included the Buddha and others.
According to the compendium of Charaka, the Charakasamhita, health and disease are
not predetermined and life may be prolonged by human effort. The compendium of
Susruta, the Susrutasamhita defines the purpose of medicine to cure the diseases of the
sick, protect the healthy, and to prolong life. Both these ancient compendia include
details of the examination, diagnosis, treatment, and prognosis of numerous ailments.
The Susrutasamhita is notable for describing procedures on various forms of surgery,
including rhinoplasty, the repair of torn ear lobes, perineal lithotomy, cataract surgery,
and several other excisions and other surgical procedures. Most remarkable is
Sushruta's penchant for scientific classification: His medical treatise consists of 184
chapters, 1,120 conditions are listed, including injuries and illnesses relating to aging
and mental illness. The Sushruta Samhita describe 125 surgical instrument, 300 surgical
procedures and classifies human surgery in 8 categories.
The Ayurveda classics mention eight branches of medicine: kayacikitsa (internal
medicine), salyacikitsa (surgery including anatomy), salakyacikitsa (eye, ear, nose, and
throat diseases), kaumarabhṛtya (pediatrics), bhutavidya (spirit medicine), and agada
tantra (toxicology), rasayana (science of rejuvenation), and vajikaraṇa (Aphrodisiac).
Apart from learning these, the student of Ayurveda was expected to know ten arts that
were indispensable in the preparation and application of his medicines: distillation,
operative skills, cooking, horticulture, metallurgy, sugar manufacture, pharmacy, analysis
and separation of minerals, compounding of metals, and preparation of alkalis. The
teaching of various subjects was done during the instruction of relevant clinical subjects.
For example, teaching of anatomy was a part of the teaching of surgery, embryology
was a part of training in pediatrics and obstetrics, and the knowledge of physiology and
pathology was interwoven in the teaching of all the clinical disciplines. The normal length
of the student's training appears to have been seven years. But the physician was to
continue to learn.
As an alternative form of medicine in India, Unani medicine got deep roots and royal
patronage during medieval times. It progressed during Indian sultanate and Mughal
periods. Unani medicine is very close to Ayurveda. Both are based on theory of the
presence of the elements in the human body. According to followers of Unani medicine,
these elements are present in different fluids and their balance leads to health and their
imbalance leads to illness.
By the 18th century A.D., Sanskrit medical wisdom still dominated. Muslim rulers built
large hospitals in 1595 in Hyderabad, and in Delhi in 1719, and numerous commentaries
on ancient texts were written.
Type: MCQ Single    |    Marks: +1/-0   |    Topic: Verbal > Reading Comprehension    |  
Time Spent: 
5 Mins 29 Secs
21. 
What is the main idea of the passage?
A. 
The history of medicine in Ancient Egypt and India.
B. 
Difference between Unani medicine and Ayurveda.
C. 
The various books that are written are discussed above.
D. 
Comparison between the ancient and present medicines.

Status :  

Correct
Answer : A
View Solution
Option (A) is correct because this can be inferred as the main idea from the above
passage. Option (B) is incorrect because this is not the main idea of the passage but is a
part of the passage. Option (C) is incorrect because no such description of the books
has been done in the passage. Option (D) is incorrect because no comparison can be
seen between the medicines. 
Bookmark Share question feedback

Passage :
Ancient Egypt developed a large, varied and fruitful medical tradition. Herodotus
described the Egyptians as "the healthiest of all men, next to the Libyans", because of
the dry climate and the notable public health system that they possessed. According to
him, "the practice of medicine is so specialized among them that each physician is a
healer of one disease and no more." Although Egyptian medicine, to a good extent, dealt
with the supernatural, it eventually developed a practical use in the fields of anatomy,
public health, and clinical diagnostics.
The Atharvaveda, a sacred text of Hinduism dating from the Early Iron Age, is one of the
first Indian text dealing with medicine, like the medicine of the Ancient Near East based
on concepts of the exorcism of demons and magic. The Atharvaveda also contain
prescriptions of herbs for various ailments. The use of herbs to treat ailments would later
form a large part of Ayurveda.
Ayurveda, meaning the "complete knowledge for long life" is another medical system of
India. Its two most famous texts belong to the schools of Charaka and Sushruta. The
earliest foundations of Ayurveda were built on a synthesis of traditional herbal practices
together with a massive addition of theoretical conceptualizations, new nosologies and
new therapies dating from about 600 BCE onwards, and coming out of the communities
of thinkers who included the Buddha and others.
According to the compendium of Charaka, the Charakasamhita, health and disease are
not predetermined and life may be prolonged by human effort. The compendium of
Susruta, the Susrutasamhita defines the purpose of medicine to cure the diseases of the
sick, protect the healthy, and to prolong life. Both these ancient compendia include
details of the examination, diagnosis, treatment, and prognosis of numerous ailments.
The Susrutasamhita is notable for describing procedures on various forms of surgery,
including rhinoplasty, the repair of torn ear lobes, perineal lithotomy, cataract surgery,
and several other excisions and other surgical procedures. Most remarkable is
Sushruta's penchant for scientific classification: His medical treatise consists of 184
chapters, 1,120 conditions are listed, including injuries and illnesses relating to aging
and mental illness. The Sushruta Samhita describe 125 surgical instrument, 300 surgical
procedures and classifies human surgery in 8 categories.
The Ayurveda classics mention eight branches of medicine: kayacikitsa (internal
medicine), salyacikitsa (surgery including anatomy), salakyacikitsa (eye, ear, nose, and
throat diseases), kaumarabhṛtya (pediatrics), bhutavidya (spirit medicine), and agada
tantra (toxicology), rasayana (science of rejuvenation), and vajikaraṇa (Aphrodisiac).
Apart from learning these, the student of Ayurveda was expected to know ten arts that
were indispensable in the preparation and application of his medicines: distillation,
operative skills, cooking, horticulture, metallurgy, sugar manufacture, pharmacy, analysis
and separation of minerals, compounding of metals, and preparation of alkalis. The
teaching of various subjects was done during the instruction of relevant clinical subjects.
For example, teaching of anatomy was a part of the teaching of surgery, embryology
was a part of training in pediatrics and obstetrics, and the knowledge of physiology and
pathology was interwoven in the teaching of all the clinical disciplines. The normal length
of the student's training appears to have been seven years. But the physician was to
continue to learn.
As an alternative form of medicine in India, Unani medicine got deep roots and royal
patronage during medieval times. It progressed during Indian sultanate and Mughal
periods. Unani medicine is very close to Ayurveda. Both are based on theory of the
presence of the elements in the human body. According to followers of Unani medicine,
these elements are present in different fluids and their balance leads to health and their
imbalance leads to illness.
By the 18th century A.D., Sanskrit medical wisdom still dominated. Muslim rulers built
large hospitals in 1595 in Hyderabad, and in Delhi in 1719, and numerous commentaries
on ancient texts were written.
Type: MCQ Single    |    Marks: +1/-0   |    Topic: Verbal > Reading Comprehension    |  
Time Spent: 
0 Mins 58 Secs
22. 
Why does the author use the term “sacred” for Atharvaveda?
A. 
Because it is a holy book for medicines.
B. 
Because it is considered as the oldest and the only book dealing with medicine.
C. 
Because it is a blessed book for medicine.
D. 
Because it is a desecrated book considered by the ancient Indian.

Status :  
Incorrect
Given Answer : A
Answer : B
View Solution
Option (B) is correct because it is mentioned in the above passage that “The
Atharvaveda, a sacred text of Hinduism dating from the Early Iron Age, is one of the first
Indian text dealing with medicine, like the medicine of the Ancient Near East based on
concepts of the exorcism of demons and magic”. Option (A) is incorrect because it does
not have the right reason. Option (C) is incorrect because it does not have the right
meaning for the term used in the passage. Option (D) is incorrect because the word
desecrated means dishonoured which is opposite meaning as per the term has used in
the passage. 
Bookmark Share question feedback

Passage :
Ancient Egypt developed a large, varied and fruitful medical tradition. Herodotus
described the Egyptians as "the healthiest of all men, next to the Libyans", because of
the dry climate and the notable public health system that they possessed. According to
him, "the practice of medicine is so specialized among them that each physician is a
healer of one disease and no more." Although Egyptian medicine, to a good extent, dealt
with the supernatural, it eventually developed a practical use in the fields of anatomy,
public health, and clinical diagnostics.
The Atharvaveda, a sacred text of Hinduism dating from the Early Iron Age, is one of the
first Indian text dealing with medicine, like the medicine of the Ancient Near East based
on concepts of the exorcism of demons and magic. The Atharvaveda also contain
prescriptions of herbs for various ailments. The use of herbs to treat ailments would later
form a large part of Ayurveda.
Ayurveda, meaning the "complete knowledge for long life" is another medical system of
India. Its two most famous texts belong to the schools of Charaka and Sushruta. The
earliest foundations of Ayurveda were built on a synthesis of traditional herbal practices
together with a massive addition of theoretical conceptualizations, new nosologies and
new therapies dating from about 600 BCE onwards, and coming out of the communities
of thinkers who included the Buddha and others.
According to the compendium of Charaka, the Charakasamhita, health and disease are
not predetermined and life may be prolonged by human effort. The compendium of
Susruta, the Susrutasamhita defines the purpose of medicine to cure the diseases of the
sick, protect the healthy, and to prolong life. Both these ancient compendia include
details of the examination, diagnosis, treatment, and prognosis of numerous ailments.
The Susrutasamhita is notable for describing procedures on various forms of surgery,
including rhinoplasty, the repair of torn ear lobes, perineal lithotomy, cataract surgery,
and several other excisions and other surgical procedures. Most remarkable is
Sushruta's penchant for scientific classification: His medical treatise consists of 184
chapters, 1,120 conditions are listed, including injuries and illnesses relating to aging
and mental illness. The Sushruta Samhita describe 125 surgical instrument, 300 surgical
procedures and classifies human surgery in 8 categories.
The Ayurveda classics mention eight branches of medicine: kayacikitsa (internal
medicine), salyacikitsa (surgery including anatomy), salakyacikitsa (eye, ear, nose, and
throat diseases), kaumarabhṛtya (pediatrics), bhutavidya (spirit medicine), and agada
tantra (toxicology), rasayana (science of rejuvenation), and vajikaraṇa (Aphrodisiac).
Apart from learning these, the student of Ayurveda was expected to know ten arts that
were indispensable in the preparation and application of his medicines: distillation,
operative skills, cooking, horticulture, metallurgy, sugar manufacture, pharmacy, analysis
and separation of minerals, compounding of metals, and preparation of alkalis. The
teaching of various subjects was done during the instruction of relevant clinical subjects.
For example, teaching of anatomy was a part of the teaching of surgery, embryology
was a part of training in pediatrics and obstetrics, and the knowledge of physiology and
pathology was interwoven in the teaching of all the clinical disciplines. The normal length
of the student's training appears to have been seven years. But the physician was to
continue to learn.
As an alternative form of medicine in India, Unani medicine got deep roots and royal
patronage during medieval times. It progressed during Indian sultanate and Mughal
periods. Unani medicine is very close to Ayurveda. Both are based on theory of the
presence of the elements in the human body. According to followers of Unani medicine,
these elements are present in different fluids and their balance leads to health and their
imbalance leads to illness.
By the 18th century A.D., Sanskrit medical wisdom still dominated. Muslim rulers built
large hospitals in 1595 in Hyderabad, and in Delhi in 1719, and numerous commentaries
on ancient texts were written.
Type: MCQ Single    |    Marks: +1/-0   |    Topic: Verbal > Reading Comprehension    |  
Time Spent: 
1 Mins 23 Secs
23. 
Which of the following shows why the Ayurveda has complete knowledge?
A. 
By the 18th century A.D. Sanskrit medical wisdom still dominated.
B. 
As it is originally from India hence it has complete knowledge from the Vedas.
C. 
The student has to learn all the eight branches as well as ten arts that were
indispensable in the preparation and application of his medicines.
D. 
The compendium of Susruta defines the purpose of medicine to cure the diseases of the
sick, protect the healthy, and to prolong life.

Status :  

Incorrect
Given Answer : A
Answer : C
View Solution
Option (C) is correct as the author has mentioned above that the students need to have
a complete knowledge about the illness as well as the medicines to make and even
apply them. Option (A) is incorrect because it does not shows any relation with the
question. Option (B) is incorrect because the very first line mentions that “Ancient Egypt
developed a large, varied and fruitful medical tradition”. Option (D) is incorrect because
this option describes a term Susruta. 
Bookmark Share question feedback

Passage :
Ancient Egypt developed a large, varied and fruitful medical tradition. Herodotus
described the Egyptians as "the healthiest of all men, next to the Libyans", because of
the dry climate and the notable public health system that they possessed. According to
him, "the practice of medicine is so specialized among them that each physician is a
healer of one disease and no more." Although Egyptian medicine, to a good extent, dealt
with the supernatural, it eventually developed a practical use in the fields of anatomy,
public health, and clinical diagnostics.
The Atharvaveda, a sacred text of Hinduism dating from the Early Iron Age, is one of the
first Indian text dealing with medicine, like the medicine of the Ancient Near East based
on concepts of the exorcism of demons and magic. The Atharvaveda also contain
prescriptions of herbs for various ailments. The use of herbs to treat ailments would later
form a large part of Ayurveda.
Ayurveda, meaning the "complete knowledge for long life" is another medical system of
India. Its two most famous texts belong to the schools of Charaka and Sushruta. The
earliest foundations of Ayurveda were built on a synthesis of traditional herbal practices
together with a massive addition of theoretical conceptualizations, new nosologies and
new therapies dating from about 600 BCE onwards, and coming out of the communities
of thinkers who included the Buddha and others.
According to the compendium of Charaka, the Charakasamhita, health and disease are
not predetermined and life may be prolonged by human effort. The compendium of
Susruta, the Susrutasamhita defines the purpose of medicine to cure the diseases of the
sick, protect the healthy, and to prolong life. Both these ancient compendia include
details of the examination, diagnosis, treatment, and prognosis of numerous ailments.
The Susrutasamhita is notable for describing procedures on various forms of surgery,
including rhinoplasty, the repair of torn ear lobes, perineal lithotomy, cataract surgery,
and several other excisions and other surgical procedures. Most remarkable is
Sushruta's penchant for scientific classification: His medical treatise consists of 184
chapters, 1,120 conditions are listed, including injuries and illnesses relating to aging
and mental illness. The Sushruta Samhita describe 125 surgical instrument, 300 surgical
procedures and classifies human surgery in 8 categories.
The Ayurveda classics mention eight branches of medicine: kayacikitsa (internal
medicine), salyacikitsa (surgery including anatomy), salakyacikitsa (eye, ear, nose, and
throat diseases), kaumarabhṛtya (pediatrics), bhutavidya (spirit medicine), and agada
tantra (toxicology), rasayana (science of rejuvenation), and vajikaraṇa (Aphrodisiac).
Apart from learning these, the student of Ayurveda was expected to know ten arts that
were indispensable in the preparation and application of his medicines: distillation,
operative skills, cooking, horticulture, metallurgy, sugar manufacture, pharmacy, analysis
and separation of minerals, compounding of metals, and preparation of alkalis. The
teaching of various subjects was done during the instruction of relevant clinical subjects.
For example, teaching of anatomy was a part of the teaching of surgery, embryology
was a part of training in pediatrics and obstetrics, and the knowledge of physiology and
pathology was interwoven in the teaching of all the clinical disciplines. The normal length
of the student's training appears to have been seven years. But the physician was to
continue to learn.
As an alternative form of medicine in India, Unani medicine got deep roots and royal
patronage during medieval times. It progressed during Indian sultanate and Mughal
periods. Unani medicine is very close to Ayurveda. Both are based on theory of the
presence of the elements in the human body. According to followers of Unani medicine,
these elements are present in different fluids and their balance leads to health and their
imbalance leads to illness.
By the 18th century A.D., Sanskrit medical wisdom still dominated. Muslim rulers built
large hospitals in 1595 in Hyderabad, and in Delhi in 1719, and numerous commentaries
on ancient texts were written.
Type: MCQ Single    |    Marks: +1/-0   |    Topic: Verbal > Reading Comprehension    |  
Time Spent: 
0 Mins 32 Secs
24. 
The author is likely to agree with which of the following questions?
A. 
The Ayurveda has its origin from the Ancient Egypt.
B. 
The Ayurveda is losing its importance in the present world.
C. 
Were the Unani medicine and royal patronage formed before Ayurveda’s?
D. 
Does the study of the Ayurveda take more than seven years?

Status :  

Correct
Answer : D
View Solution
Option (D) is correct because it is mentioned in the above passage that “The normal
length of the student's training appears to have been seven years. But the physician was
to continue to learn”. Option (A) is incorrect because it can be inferred from the passage
that Ayurveda has its roots in India. Option (B) is incorrect because no information can
be seen in the passage hence the author cannot agree with the option. Option (C) is
incorrect because it is mentioned in the passage that “Unani medicine is very close to
Ayurveda” which shows that they are not formed before Ayurveda’s. 
Bookmark Share question feedback

Passage :
Ancient Egypt developed a large, varied and fruitful medical tradition. Herodotus
described the Egyptians as "the healthiest of all men, next to the Libyans", because of
the dry climate and the notable public health system that they possessed. According to
him, "the practice of medicine is so specialized among them that each physician is a
healer of one disease and no more." Although Egyptian medicine, to a good extent, dealt
with the supernatural, it eventually developed a practical use in the fields of anatomy,
public health, and clinical diagnostics.
The Atharvaveda, a sacred text of Hinduism dating from the Early Iron Age, is one of the
first Indian text dealing with medicine, like the medicine of the Ancient Near East based
on concepts of the exorcism of demons and magic. The Atharvaveda also contain
prescriptions of herbs for various ailments. The use of herbs to treat ailments would later
form a large part of Ayurveda.
Ayurveda, meaning the "complete knowledge for long life" is another medical system of
India. Its two most famous texts belong to the schools of Charaka and Sushruta. The
earliest foundations of Ayurveda were built on a synthesis of traditional herbal practices
together with a massive addition of theoretical conceptualizations, new nosologies and
new therapies dating from about 600 BCE onwards, and coming out of the communities
of thinkers who included the Buddha and others.
According to the compendium of Charaka, the Charakasamhita, health and disease are
not predetermined and life may be prolonged by human effort. The compendium of
Susruta, the Susrutasamhita defines the purpose of medicine to cure the diseases of the
sick, protect the healthy, and to prolong life. Both these ancient compendia include
details of the examination, diagnosis, treatment, and prognosis of numerous ailments.
The Susrutasamhita is notable for describing procedures on various forms of surgery,
including rhinoplasty, the repair of torn ear lobes, perineal lithotomy, cataract surgery,
and several other excisions and other surgical procedures. Most remarkable is
Sushruta's penchant for scientific classification: His medical treatise consists of 184
chapters, 1,120 conditions are listed, including injuries and illnesses relating to aging
and mental illness. The Sushruta Samhita describe 125 surgical instrument, 300 surgical
procedures and classifies human surgery in 8 categories.
The Ayurveda classics mention eight branches of medicine: kayacikitsa (internal
medicine), salyacikitsa (surgery including anatomy), salakyacikitsa (eye, ear, nose, and
throat diseases), kaumarabhṛtya (pediatrics), bhutavidya (spirit medicine), and agada
tantra (toxicology), rasayana (science of rejuvenation), and vajikaraṇa (Aphrodisiac).
Apart from learning these, the student of Ayurveda was expected to know ten arts that
were indispensable in the preparation and application of his medicines: distillation,
operative skills, cooking, horticulture, metallurgy, sugar manufacture, pharmacy, analysis
and separation of minerals, compounding of metals, and preparation of alkalis. The
teaching of various subjects was done during the instruction of relevant clinical subjects.
For example, teaching of anatomy was a part of the teaching of surgery, embryology
was a part of training in pediatrics and obstetrics, and the knowledge of physiology and
pathology was interwoven in the teaching of all the clinical disciplines. The normal length
of the student's training appears to have been seven years. But the physician was to
continue to learn.
As an alternative form of medicine in India, Unani medicine got deep roots and royal
patronage during medieval times. It progressed during Indian sultanate and Mughal
periods. Unani medicine is very close to Ayurveda. Both are based on theory of the
presence of the elements in the human body. According to followers of Unani medicine,
these elements are present in different fluids and their balance leads to health and their
imbalance leads to illness.
By the 18th century A.D., Sanskrit medical wisdom still dominated. Muslim rulers built
large hospitals in 1595 in Hyderabad, and in Delhi in 1719, and numerous commentaries
on ancient texts were written.
Type: MCQ Single    |    Marks: +1/-0   |    Topic: Verbal > Reading Comprehension    |  
Time Spent: 
1 Mins 36 Secs
25. 
Which of the following does the author NOT agree with?
A. 
Unani medicine is very close to Ayurveda. Both are based on theory of the presence of
the elements in the human body.
B. 
The normal length of the student's training appears to have been seven years.
C. 
Medical treatise consists of 184 chapters, 1,120 conditions are listed, including injuries
and illnesses relating to aging and mental illness.
D. 
The Atharvaveda, are not the one of the first Indian text dealing with medicine.

Status :  

Incorrect
Given Answer : B
Answer : D
View Solution
Option (D) is correct because it is mentioned in the passage that “The Atharvaveda, a
sacred text of Hinduism dating from the Early Iron Age, is one of the first Indian text
dealing with medicine, as the medicine of the Ancient Near East based on concepts of
the exorcism of demons and magic”. Option (A) is incorrect because it is already
mentioned in the passage hence the author will agree with the option. Option (B) is
incorrect because the information is provided by the author in the passage. Option (C) is
incorrect because it is already mentioned in the passage hence the author does not
have any reason to disagree with the option. 
Bookmark Share question feedback

Passage :
In Europe, before the Enlightenment of the eighteenth and nineteenth century, education
was the responsibility of parents and the church. With the French and American
Revolution education was established also as a public function. It was thought that the
state, by assuming a more active role in the sphere of education, could help to make
education available and accessible to all. Education had thus far been primarily available
to the upper social classes and public education was perceived as a means of realising
the egalitarian ideals underlining both revolutions.
However, neither the American Declaration of Independence (1776) nor the French
Declaration of the Rights of Man (1789) protected the right to education as the liberal
concepts of human rights in the nineteenth century envisaged that parents retained the
primary duty for providing education to their children. It was the states obligation to
ensure that parents complied with this duty, and many states enacted legislation making
school attendance compulsory. Furthermore, child labour laws were enacted to limit the
number of hours per day children could be employed, to ensure children would attend
school. States also became involved in the legal regulation of curricula and established
minimum educational standards.
In On Liberty John Stuart Mill wrote that an "education established and controlled by the
State should only exist, if it exists at all, as one among many competing experiments,
carried on for the purpose of example and stimulus to keep the others up to a certain
standard of excellence." Liberal thinkers of the nineteenth century pointed to the dangers
to too much state involvement in the sphere of education, but relied on state intervention
to reduce the dominance of the church, and to protect the right to education of children
against their own parents. In the latter half of the nineteenth century, educational rights
were included in domestic bills of rights. The 1849 Paulskirchenverfassung, the
constitution of the German Empire, strongly influenced subsequent European
constitutions and devoted Article 152 to 158 of its bill of rights to education. The
constitution recognised education as a function of the state, independent of the church.
Remarkable at the time, the constitution proclaimed the right to free education for the
poor, but the constitution did not explicitly require the state to set up educational
institutions. Instead the constitution protected the rights of citizens to found and operate
schools and to provide home education. The constitution also provided for freedom of
science and teaching, and it guaranteed the right of everybody to choose a vocation and
train for it.
The nineteenth century also saw the development of socialist theory, which held that the
primary task of the state was to ensure the economic and social well-being of the
community through government intervention and regulation. Socialist theory recognised
that individuals had claims to basic welfare services against the state and education was
viewed as one of these welfare entitlements. This was in contrast to liberal theory at the
time, which regarded non-state actors as the prime providers of education. Socialist
ideals were enshrined in the 1936 Soviet Constitution, which was the first constitution to
recognise the right to education with a corresponding obligation of the state to provide
such education. The constitution guaranteed free and compulsory education at all levels,
a system of state scholarships and vocational training in state enterprises. Subsequently
the right to education featured strongly in the constitutions of socialist states. As a
political goal, right to education was declared in F. D. Roosevelt's 1944 speech on the
Second Bill of Rights.
Type: MCQ Single    |    Marks: +1/-0   |    Topic: Verbal > Reading Comprehension    |  
Time Spent: 
6 Mins 17 Secs
26. 
What is the main idea of the passage?
A. 
To show how Right of education became a responsibility of each state.
B. 
To show diverse cultural and socio-economic factors.
C. 
To show the problems and causes for education.
D. 
Definition of Right to Education.

Status :  

Incorrect
Given Answer : D
Answer : A
View Solution
Option (A) is correct because the author in the above passage has mentioned that “It
was the state's obligation to ensure that parents complied with this duty, and many
states enacted legislation making school attendance compulsory”. Option (B) is incorrect
because no information is provided in the passage about diverse cultural and socio-
economic factors. Option (C) is incorrect because no information about the problems
and causes for education can be seen in the passage. Option (D) is incorrect because
no definition can be seen in the passage. 
Bookmark Share question feedback

Passage :
In Europe, before the Enlightenment of the eighteenth and nineteenth century, education
was the responsibility of parents and the church. With the French and American
Revolution education was established also as a public function. It was thought that the
state, by assuming a more active role in the sphere of education, could help to make
education available and accessible to all. Education had thus far been primarily available
to the upper social classes and public education was perceived as a means of realising
the egalitarian ideals underlining both revolutions.
However, neither the American Declaration of Independence (1776) nor the French
Declaration of the Rights of Man (1789) protected the right to education as the liberal
concepts of human rights in the nineteenth century envisaged that parents retained the
primary duty for providing education to their children. It was the states obligation to
ensure that parents complied with this duty, and many states enacted legislation making
school attendance compulsory. Furthermore, child labour laws were enacted to limit the
number of hours per day children could be employed, to ensure children would attend
school. States also became involved in the legal regulation of curricula and established
minimum educational standards.
In On Liberty John Stuart Mill wrote that an "education established and controlled by the
State should only exist, if it exists at all, as one among many competing experiments,
carried on for the purpose of example and stimulus to keep the others up to a certain
standard of excellence." Liberal thinkers of the nineteenth century pointed to the dangers
to too much state involvement in the sphere of education, but relied on state intervention
to reduce the dominance of the church, and to protect the right to education of children
against their own parents. In the latter half of the nineteenth century, educational rights
were included in domestic bills of rights. The 1849 Paulskirchenverfassung, the
constitution of the German Empire, strongly influenced subsequent European
constitutions and devoted Article 152 to 158 of its bill of rights to education. The
constitution recognised education as a function of the state, independent of the church.
Remarkable at the time, the constitution proclaimed the right to free education for the
poor, but the constitution did not explicitly require the state to set up educational
institutions. Instead the constitution protected the rights of citizens to found and operate
schools and to provide home education. The constitution also provided for freedom of
science and teaching, and it guaranteed the right of everybody to choose a vocation and
train for it.
The nineteenth century also saw the development of socialist theory, which held that the
primary task of the state was to ensure the economic and social well-being of the
community through government intervention and regulation. Socialist theory recognised
that individuals had claims to basic welfare services against the state and education was
viewed as one of these welfare entitlements. This was in contrast to liberal theory at the
time, which regarded non-state actors as the prime providers of education. Socialist
ideals were enshrined in the 1936 Soviet Constitution, which was the first constitution to
recognise the right to education with a corresponding obligation of the state to provide
such education. The constitution guaranteed free and compulsory education at all levels,
a system of state scholarships and vocational training in state enterprises. Subsequently
the right to education featured strongly in the constitutions of socialist states. As a
political goal, right to education was declared in F. D. Roosevelt's 1944 speech on the
Second Bill of Rights.
Type: MCQ Single    |    Marks: +1/-0   |    Topic: Verbal > Reading Comprehension    |  
Time Spent: 
0 Mins 11 Secs
27. 
Why does the author use the term “Enlightenment”?
A. 
it means obfuscation.
B. 
it means education.
C. 
it means illumination.
D. 
it means ignorance.

Status :  

Correct
Answer : B
View Solution
Option (B) is correct because it is mentioned rightly as per the passage. Option (A) is
incorrect because the word obfuscation means complication and it is an opposite word
for the term used. Option (C) is incorrect because the word illumination is used in terms
of light which is not right for the term. Option (D) is incorrect because the word ignorance
means unawareness which is a opposite word for the term. 
Bookmark Share question feedback

Passage :
In Europe, before the Enlightenment of the eighteenth and nineteenth century, education
was the responsibility of parents and the church. With the French and American
Revolution education was established also as a public function. It was thought that the
state, by assuming a more active role in the sphere of education, could help to make
education available and accessible to all. Education had thus far been primarily available
to the upper social classes and public education was perceived as a means of realising
the egalitarian ideals underlining both revolutions.
However, neither the American Declaration of Independence (1776) nor the French
Declaration of the Rights of Man (1789) protected the right to education as the liberal
concepts of human rights in the nineteenth century envisaged that parents retained the
primary duty for providing education to their children. It was the states obligation to
ensure that parents complied with this duty, and many states enacted legislation making
school attendance compulsory. Furthermore, child labour laws were enacted to limit the
number of hours per day children could be employed, to ensure children would attend
school. States also became involved in the legal regulation of curricula and established
minimum educational standards.
In On Liberty John Stuart Mill wrote that an "education established and controlled by the
State should only exist, if it exists at all, as one among many competing experiments,
carried on for the purpose of example and stimulus to keep the others up to a certain
standard of excellence." Liberal thinkers of the nineteenth century pointed to the dangers
to too much state involvement in the sphere of education, but relied on state intervention
to reduce the dominance of the church, and to protect the right to education of children
against their own parents. In the latter half of the nineteenth century, educational rights
were included in domestic bills of rights. The 1849 Paulskirchenverfassung, the
constitution of the German Empire, strongly influenced subsequent European
constitutions and devoted Article 152 to 158 of its bill of rights to education. The
constitution recognised education as a function of the state, independent of the church.
Remarkable at the time, the constitution proclaimed the right to free education for the
poor, but the constitution did not explicitly require the state to set up educational
institutions. Instead the constitution protected the rights of citizens to found and operate
schools and to provide home education. The constitution also provided for freedom of
science and teaching, and it guaranteed the right of everybody to choose a vocation and
train for it.
The nineteenth century also saw the development of socialist theory, which held that the
primary task of the state was to ensure the economic and social well-being of the
community through government intervention and regulation. Socialist theory recognised
that individuals had claims to basic welfare services against the state and education was
viewed as one of these welfare entitlements. This was in contrast to liberal theory at the
time, which regarded non-state actors as the prime providers of education. Socialist
ideals were enshrined in the 1936 Soviet Constitution, which was the first constitution to
recognise the right to education with a corresponding obligation of the state to provide
such education. The constitution guaranteed free and compulsory education at all levels,
a system of state scholarships and vocational training in state enterprises. Subsequently
the right to education featured strongly in the constitutions of socialist states. As a
political goal, right to education was declared in F. D. Roosevelt's 1944 speech on the
Second Bill of Rights.
Type: MCQ Single    |    Marks: +1/-0   |    Topic: Verbal > Reading Comprehension    |  
Time Spent: 
0 Mins 21 Secs
28. 
Which of the following exemplifies that socialist theory was important?
A. 
As education was the responsibility of parents and the church.
B. 
As it created awareness in the society about education.
C. 
As the economic and social well-being were taken care of by the government
intervention and regulation.
D. 
It helped the secondary education also to be compulsory for the children.

Status :  

Correct
Answer : C
View Solution
Option (C) is correct as it is mentioned in the passage that “The nineteenth century also
saw the development of socialist theory, which held that the primary task of the state
was to ensure the economic and social well-being of the community through government
intervention and regulation”. Option (A) is incorrect because the option is not related to
the question. Option (B) is incorrect because it did not create any awareness in the
society. Option (D) is incorrect because no information about the secondary education is
mentioned in the passage. 
Bookmark Share question feedback

Passage :
In Europe, before the Enlightenment of the eighteenth and nineteenth century, education
was the responsibility of parents and the church. With the French and American
Revolution education was established also as a public function. It was thought that the
state, by assuming a more active role in the sphere of education, could help to make
education available and accessible to all. Education had thus far been primarily available
to the upper social classes and public education was perceived as a means of realising
the egalitarian ideals underlining both revolutions.
However, neither the American Declaration of Independence (1776) nor the French
Declaration of the Rights of Man (1789) protected the right to education as the liberal
concepts of human rights in the nineteenth century envisaged that parents retained the
primary duty for providing education to their children. It was the states obligation to
ensure that parents complied with this duty, and many states enacted legislation making
school attendance compulsory. Furthermore, child labour laws were enacted to limit the
number of hours per day children could be employed, to ensure children would attend
school. States also became involved in the legal regulation of curricula and established
minimum educational standards.
In On Liberty John Stuart Mill wrote that an "education established and controlled by the
State should only exist, if it exists at all, as one among many competing experiments,
carried on for the purpose of example and stimulus to keep the others up to a certain
standard of excellence." Liberal thinkers of the nineteenth century pointed to the dangers
to too much state involvement in the sphere of education, but relied on state intervention
to reduce the dominance of the church, and to protect the right to education of children
against their own parents. In the latter half of the nineteenth century, educational rights
were included in domestic bills of rights. The 1849 Paulskirchenverfassung, the
constitution of the German Empire, strongly influenced subsequent European
constitutions and devoted Article 152 to 158 of its bill of rights to education. The
constitution recognised education as a function of the state, independent of the church.
Remarkable at the time, the constitution proclaimed the right to free education for the
poor, but the constitution did not explicitly require the state to set up educational
institutions. Instead the constitution protected the rights of citizens to found and operate
schools and to provide home education. The constitution also provided for freedom of
science and teaching, and it guaranteed the right of everybody to choose a vocation and
train for it.
The nineteenth century also saw the development of socialist theory, which held that the
primary task of the state was to ensure the economic and social well-being of the
community through government intervention and regulation. Socialist theory recognised
that individuals had claims to basic welfare services against the state and education was
viewed as one of these welfare entitlements. This was in contrast to liberal theory at the
time, which regarded non-state actors as the prime providers of education. Socialist
ideals were enshrined in the 1936 Soviet Constitution, which was the first constitution to
recognise the right to education with a corresponding obligation of the state to provide
such education. The constitution guaranteed free and compulsory education at all levels,
a system of state scholarships and vocational training in state enterprises. Subsequently
the right to education featured strongly in the constitutions of socialist states. As a
political goal, right to education was declared in F. D. Roosevelt's 1944 speech on the
Second Bill of Rights.
Type: MCQ Single    |    Marks: +1/-0   |    Topic: Verbal > Reading Comprehension    |  
Time Spent: 
0 Mins 28 Secs
29. 
The author is likely to agree with which of the following questions?
A. 
Is the right to education a universal entitlement to education?
B. 
Is the right to education recognized in the International Covenant on Economic, Social
and Cultural Rights as a human right?
C. 
Will the realisation of the right to education on a national level be achieved through
compulsory education only?
D. 
Was the right to education declared in F. D. Roosevelt's 1944 speech on the Second Bill
of Rights?

Status :  

Correct
Answer : D
View Solution
Option (D) is correct because it is mentioned in the last line of the passage that “As a
political goal, right to education was declared in F. D. Roosevelt's 1944 speech on the
Second Bill of Rights”. Option (A) is incorrect because is not mentioned in the passage
hence the author will not agree with option. Option (B) is incorrect because no
information is provided in the passage. Option (C) is incorrect because this is not
mentioned in the passage hence the author will not agree with option. 
Bookmark Share question feedback

Passage :
In Europe, before the Enlightenment of the eighteenth and nineteenth century, education
was the responsibility of parents and the church. With the French and American
Revolution education was established also as a public function. It was thought that the
state, by assuming a more active role in the sphere of education, could help to make
education available and accessible to all. Education had thus far been primarily available
to the upper social classes and public education was perceived as a means of realising
the egalitarian ideals underlining both revolutions.
However, neither the American Declaration of Independence (1776) nor the French
Declaration of the Rights of Man (1789) protected the right to education as the liberal
concepts of human rights in the nineteenth century envisaged that parents retained the
primary duty for providing education to their children. It was the states obligation to
ensure that parents complied with this duty, and many states enacted legislation making
school attendance compulsory. Furthermore, child labour laws were enacted to limit the
number of hours per day children could be employed, to ensure children would attend
school. States also became involved in the legal regulation of curricula and established
minimum educational standards.
In On Liberty John Stuart Mill wrote that an "education established and controlled by the
State should only exist, if it exists at all, as one among many competing experiments,
carried on for the purpose of example and stimulus to keep the others up to a certain
standard of excellence." Liberal thinkers of the nineteenth century pointed to the dangers
to too much state involvement in the sphere of education, but relied on state intervention
to reduce the dominance of the church, and to protect the right to education of children
against their own parents. In the latter half of the nineteenth century, educational rights
were included in domestic bills of rights. The 1849 Paulskirchenverfassung, the
constitution of the German Empire, strongly influenced subsequent European
constitutions and devoted Article 152 to 158 of its bill of rights to education. The
constitution recognised education as a function of the state, independent of the church.
Remarkable at the time, the constitution proclaimed the right to free education for the
poor, but the constitution did not explicitly require the state to set up educational
institutions. Instead the constitution protected the rights of citizens to found and operate
schools and to provide home education. The constitution also provided for freedom of
science and teaching, and it guaranteed the right of everybody to choose a vocation and
train for it.
The nineteenth century also saw the development of socialist theory, which held that the
primary task of the state was to ensure the economic and social well-being of the
community through government intervention and regulation. Socialist theory recognised
that individuals had claims to basic welfare services against the state and education was
viewed as one of these welfare entitlements. This was in contrast to liberal theory at the
time, which regarded non-state actors as the prime providers of education. Socialist
ideals were enshrined in the 1936 Soviet Constitution, which was the first constitution to
recognise the right to education with a corresponding obligation of the state to provide
such education. The constitution guaranteed free and compulsory education at all levels,
a system of state scholarships and vocational training in state enterprises. Subsequently
the right to education featured strongly in the constitutions of socialist states. As a
political goal, right to education was declared in F. D. Roosevelt's 1944 speech on the
Second Bill of Rights.
Type: MCQ Single    |    Marks: +1/-0   |    Topic: Verbal > Reading Comprehension    |  
Time Spent: 
0 Mins 40 Secs
30. 
Which of the following does the author NOT agree with?
A. 
The constitution guaranteed free and compulsory education at all levels.
B. 
Socialist ideals were enshrined in the 1936 Soviet Constitution, which was the first
constitution to recognise the right to education.
C. 
The constitution recognised education as a function of the state, independent of the
church.
D. 
International law protects the right to pre-primary education in a state.

Status :  

Incorrect
Given Answer : C
Answer : D
View Solution
Option (D) is correct because the international laws cannot be applicable in different
states. The international laws are made on general bases. Option (A) is incorrect
because it is already mentioned in the passage that “The constitution guaranteed free
and compulsory education at all levels, a system of state scholarships and vocational
training in state enterprises”. Option (B) is incorrect because it is already mentioned in
the passage that “Socialist ideals were enshrined in the 1936 Soviet Constitution, which
was the first constitution to recognise the right to education with a corresponding
obligation of the state to provide such education”. Option (C) is incorrect because it is
already mentioned in the passage that “The 1849 Paulskirchenverfassung, the
constitution of the German Empire, strongly influenced subsequent European
constitutions and devoted Article 152 to 158 of its bill of rights to education. The
constitution recognised education as a function of the state, independent of the church”. 
Bookmark Share question feedback

Section Name : General Awareness


Type: MCQ Single    |    Marks: +1/-0   |    Topic: General Awareness > Static    |  
Time Spent: 
0 Mins 13 Secs
1. 
The annulment of partition of Bengal was done by which of the following?
A. 
Lord Hardinge
B. 
Lord Willian Bentinck
C. 
Lord Canning
D. 
Lord Lytton

Status :  

Incorrect
Given Answer : D
Answer : A
View Solution
Lord Hardinge, the Viceroy of India from 1910 to 1916, had annulled the partition of
Bengal in 1911. 
The partition of Bengal was done by Lord Curzon on 16th October and the Annullment
bof Bengal was done by Lord Hardinge in 1911.
Bookmark Share question feedback

Type: MCQ Single    |    Marks: +1/-0   |    Topic: General Awareness > Static    |  


Time Spent: 
0 Mins 14 Secs
2. 
Which of the following newspapers is NOT related to 'Madan Mohan Malaviya' ? 
A. 
Leader
B. 
Comrade
C. 
Hindustan
D. 
Abhyudaya

Status :  

Incorrect
Given Answer : C
Answer : B
View Solution
Comrade is not related to Madan Mohan Malaviya who had been served as the
president of Indian National Confress for four times. Comrade was published and edited
by Maulana Mohammad Ali between 1911 and 1914.
Bookmark Share question feedback

Type: MCQ Single    |    Marks: +1/-0   |    Topic: General Awareness > Static    |  


Time Spent: 
0 Mins 11 Secs
3. 
Which state is the largest producer of MICA in india?
A. 
Odisha
B. 
Rajasthan
C. 
Jharkhand
D. 
Andhra Pradesh

Status :  

Incorrect
Given Answer : C
Answer : D
View Solution
Andhra Pradesh is the largest producer of MICA in India followed by Rajasthan and
Jharkhand. MICA is used in paints.
Bookmark Share question feedback

Type: MCQ Single    |    Marks: +1/-0   |    Topic: General Awareness > Static    |  


Time Spent: 
0 Mins 16 Secs
4. 
Which layer of the atmosphere has the maximum concentration of Ozone gas?
A. 
Troposphere
B. 
Stratosphere
C. 
Thermosphere
D. 
Mesosphere

Status :  

Correct
Answer : B
View Solution
Stratosphere layer of atmosphere has the maximum concentration of Ozone gas.
Approximately 10 per cent of Ozone layer is present in Troposphere and rest of the 90
per cent found in Stratosphere layer of atmosphere. Ozone layer in Stratosphere
atmosphere absorbs most of the ultraviolet radiation from the Sun. Stratosphere is the
second layer of atmosphere whereas Troposphere is the lowest layer of the
atmossphere. The Thermosphere is the layer of the Earth's atmosphere is above the
Mesosphere. There are five layers of atmosphere namely - Troposphere, Stratosphere,
Mesosphere, Thermosphere and Exosphere.
Bookmark Share question feedback

Type: MCQ Single    |    Marks: +1/-0   |    Topic: General Awareness > Static    |  


Time Spent: 
0 Mins 20 Secs
5. 
Which of the following is the highest source of protein ?
A. 
Sun Flower
B. 
Soyabeans
C. 
Grams
D. 
Wheat

Status :  

Incorrect
Given Answer : C
Answer : B
View Solution
Soyabean has the highest source of protein content of 36 g, Grams contain 19g protein
content, Sun Flower protein content is 21g and wheat is 13.2g
Bookmark Share question feedback

Type: MCQ Single    |    Marks: +1/-0   |    Topic: General Awareness > Static    |  


Time Spent: 
0 Mins 13 Secs
6. 
Which country imposed the world's toughest law against plastic bags recently?
A. 
Japan 
B. 
India
C. 
Kenya
D. 
France

Status :  

Incorrect
Given Answer : A
Answer : C
View Solution
Kenya imposed the world's toughest law against plastic bags recently.
If they sell or use any plastic bags it would become a risk of imprisonment of upto four
years or fines of $40,000. 
Plastic is banned in more than 40 other countries including China, France, Rwanda and
Italy.
Bookmark Share question feedback

Type: MCQ Single    |    Marks: +1/-0   |    Topic: General Awareness > Static    |  


Time Spent: 
0 Mins 11 Secs
7. 
When was the Montreal protocol assigned to reduce production of Chlorofluorocarbons?
A. 
2001
B. 
1980
C. 
1987
D. 
1994

Status :  

Correct
Answer : C
View Solution
Montreal Protocol is an international treaty to protect the Ozone layer. It was agreed on
26th August 1987. Its main objective was phasing out the production of numerous
substances that are responsible for Ozone depletion. It came into effect on 1st January
1989. It became its signatory member on 19th June 1992. 
Bookmark Share question feedback

Type: MCQ Single    |    Marks: +1/-0   |    Topic: General Awareness > Static    |  


Time Spent: 
0 Mins 8 Secs
8. 
Which of the following countries has built an 'artificial sun'?
A. 
Japan
B. 
China
C. 
Singapore
D. 
Israel

Status :  

Correct
Answer : B
View Solution
China has built an 'artificial sun' that reaches temperature six times that of the core of
the sun.
The reactor named 'Experimental Advanced Superconducting Tokamak' (EAST) is
designed to replicate the processes of the sun as part of a project to turn hydrogen into
cost- effective green energy,
It involves fusion of hydrogen atoms together to form heavier elements, such as helium. 
Bookmark Share question feedback

Type: MCQ Single    |    Marks: +1/-0   |    Topic: General Awareness > Static    |  


Time Spent: 
0 Mins 13 Secs
9. 
According to the Indus Water treaty which river will be not governed by Pakistan?  
A. 
Indus
B. 
Chenab
C. 
Jhelum
D. 
Beas

Status :  

Incorrect
Given Answer : A
Answer : D
View Solution
The Indus Water treaty (IWT) is a water distribution treaty between India and Pakistan
which was signed on September 19, 1960.
The treaty deals with river Indus and its five tributaries.These are divided into two
categories - 
Eastern Rivers:

1. Sutlej
2. Beas
3. Ravi

Western Rivers:

1. Jhelum
2. Chenab
3. Indus

According to treaty, all the water of eastern rivers shall be available for unrestricted use
in India. India should let unrestricted flow of water from western rivers to Pakistan. 
Bookmark Share question feedback
Type: MCQ Single    |    Marks: +1/-0   |    Topic: General Awareness > Static    |  
Time Spent: 
0 Mins 15 Secs
10. 
Which one of the following is a Wartime Gallantry Award?
A. 
Shaurya Chakra 
B. 
Kirti Chakra
C. 
Yudh Seva Medal 
D. 
Param Vir Chakra

Status :  

Correct
Answer : D
View Solution
Peacetime Gallantry Award-

1. Ashok Chakra Award


2. Kirti Chakra
3. Shaurya Chakra

Wartime Gallantry Award - 

1. Param Vir Chakra


2. Maha Vir Chakra
3. Vir Chakra

Wartime Distinguished Service -

1. Sarvottam Yudh Seva Medal


2. Uttam Yudh Seva Medal
3. Yudh Seva Medal

 
Bookmark Share question feedback

Type: MCQ Single    |    Marks: +1/-0   |    Topic: General Awareness > Static    |  


Time Spent: 
0 Mins 13 Secs
11. 
'Kunitha' is a ritual dance form of which state? 
A. 
Gujarat
B. 
Bihar
C. 
Karnataka
D. 
Madhya Pradesh 

Status :  

Correct
Answer : C
View Solution
The ritual dances of Karnataka are known as Kunitha. One such dance is the Dollu
Kunitha, a popular dance form accompanied by singing and the beats of decorated
drums. This dance is primarily performed by men from the shepherd or Kuruba caste.
The Dollu Kunitha is characterized by vigorous drumbeats, quick movements and
synchronized group formations. 
Bookmark Share question feedback

Type: MCQ Single    |    Marks: +1/-0   |    Topic: General Awareness > Static    |  


Time Spent: 
0 Mins 13 Secs
12. 
The second Ozone hole was detected over which of the following?
A. 
Antartica
B. 
Australia
C. 
Arctic
D. 
Sweden

Status :  

Incorrect
Given Answer : A
Answer : C
View Solution
The second Ozone hole was detected over Arctic.
Bookmark Share question feedback

Type: MCQ Single    |    Marks: +1/-0   |    Topic: General Awareness > Static    |  


Time Spent: 
0 Mins 12 Secs
13. 
With which of the following is 'Van Mahotsav' associated?
A. 
Insurance of Trees
B. 
Genetic modification of Trees
C. 
Planting of Trees
D. 
Increase in Crops
Status :  

Correct
Answer : C
View Solution
Van Mohatsav is an annual tree- planting movement in India, which began in 1950. The
name Van Mahotsav means Festival of Forest. It was started by K.M. Munshi.
Bookmark Share question feedback

Type: MCQ Single    |    Marks: +1/-0   |    Topic: General Awareness > Static    |  


Time Spent: 
0 Mins 13 Secs
14. 
Who among the following is not a permanent member of Malabar trilateral naval
exercise?
A. 
United States of America
B. 
India 
C. 
Japan 
D. 
Vietnam

Status :  

Incorrect
Given Answer : B
Answer : D
View Solution
Exercise Malabar is a trilateral naval exercise involving the United States of America,
Japan and India as permanent partners. Originally begun in 1992 as a bilateral exercise
between India and the United States. Japan became a permanent partner in 2015. Past
non permanent participants were Australia and Singapore.
Bookmark Share question feedback

Type: MCQ Single    |    Marks: +1/-0   |    Topic: General Awareness > Static    |  


Time Spent: 
0 Mins 11 Secs
15. 
Who among the following was a classical Indian flute player?
A. 
Bismillah Khan
B. 
Hariprasad Chaurasia
C. 
Zakir Hussain
D. 
Shiv Kumar Sharma

Status :  
Incorrect
Given Answer : A
Answer : B
View Solution
Artists Instruments
Bismillah Khan Shehnai
Hariprasad Chaurasia Flute
Zakir Hussain Tabla 
Shiv Kumar Sharma Santoor

 
Bookmark Share question feedback

Type: MCQ Single    |    Marks: +1/-0   |    Topic: General Awareness > Static    |  


Time Spent: 
0 Mins 9 Secs
16. 
Which of the following is capital of Cambodia?
A. 
Nicosia
B. 
Hanoi
C. 
Dili
D. 
Phnom Penh

Status :  

Incorrect
Given Answer : B
Answer : D
View Solution
Capital of Cambodia Phnom Penh
Currency Cambodian Riel
King Norodom Sihamoni
Prime Minister Hun Sen
Official Language Khmer

 
Bookmark Share question feedback

Type: MCQ Single    |    Marks: +1/-0   |    Topic: General Awareness > Static    |  


Time Spent: 
0 Mins 17 Secs
17. 
Which one of the following Vedas dealt with rituals?
A. 
Rig Veda 
B. 
Sama Veda 
C. 
Yajur Veda
D. 
Atharva Veda

Status :  

Incorrect
Given Answer : A
Answer : C
View Solution
There are four Vedas : -

1. Rig Veda 
2. Sama Veda
3. Yajur Veda
4. Atharva Veda

Rig Veda is a collection of hymns dedicated to various deities.


The Sama Veda deals with melodies.
The Yajur Veda contains the rituals of the Yagna.
The Atharva Veda contains magic spells.
Bookmark Share question feedback

Type: MCQ Single    |    Marks: +1/-0   |    Topic: General Awareness > Static    |  


Time Spent: 
0 Mins 9 Secs
18. 
How many schedules are there in our Constitution ?
A. 
10
B. 
8
C. 
12
D. 
11

Status :  

Correct
Answer : C
View Solution
The Indian Constitution has 12 schedules  and 448 articles  written in 25 parts. Also, 103
amendments have been made in it till now. It was also the world's lengthiest written
constitution at the time of its commencement with 8 schedules and 395 articles in 22
parts.
Bookmark Share question feedback

Type: MCQ Single    |    Marks: +1/-0   |    Topic: General Awareness > Static    |  


Time Spent: 
0 Mins 19 Secs
19. 
The Kishtwar National park (KNP) is located in which state? 
A. 
Jammu and Kashmir
B. 
Himachal Pradesh
C. 
Punjab
D. 
Sikkim

Status :  

Correct
Answer : A
View Solution
Kishtwar National Park (KNP) is located in the Kishtwar district of Jammu and Kashmir,
India. This national Park was made to protect the endangered species of snow leopards
and was given a status of a national park in 1981.
Bookmark Share question feedback

Type: MCQ Single    |    Marks: +1/-0   |    Topic: General Awareness > Static    |  


Time Spent: 
0 Mins 10 Secs
20. 
The concept of single citizenship in the Indian Constitution is inspired from which
country? 
A. 
USA
B. 
France 
C. 
Ireland
D. 
England

Status :  

Incorrect
Given Answer : C
Answer : D
View Solution
Though the Indian Constitution is federal and envisages dual polity, it provides for only
single citizenship unlike USA that has dual citizenship. This feature has been adopted
from British Constitution.
Bookmark Share question feedback

Type: MCQ Single    |    Marks: +1/-0   |    Topic: General Awareness > Static    |  


Time Spent: 
0 Mins 10 Secs
21. 
What is the shape of the Atlantic Ocean?
A. 
Almost Circular
B. 
'S' Shaped 
C. 
Almost Triangular
D. 
None of these

Status :  

Incorrect
Given Answer : D
Answer : B
View Solution
The shape of the Atlantic Ocean is of an 'S' shaped basin.
It extends between Europe and Africa to the east and the America's to the west.
The Atlantic Ocean is the second largest ocean in the World after Pacific Ocean.
Bookmark Share question feedback

Type: MCQ Single    |    Marks: +1/-0   |    Topic: General Awareness > Static    |  


Time Spent: 
0 Mins 10 Secs
22. 
Which country has withdrawn from UN-backed anti-corruption commission?
A. 
Ukraine
B. 
Uganda
C. 
Guatemala
D. 
Netherlands

Status :  

Incorrect
Given Answer : B
Answer : C
View Solution
Guatemala has said it is withdrawing from a UN- backed anti corruption commission.
President Jimmy Morales accused the International Commission Against Impunity in
Guatemala, commonly known as CICIG, of polarising the country and putting its security
at risk.
Bookmark Share question feedback

Type: MCQ Single    |    Marks: +1/-0   |    Topic: General Awareness > Static    |  


Time Spent: 
0 Mins 14 Secs
23. 
Which of the following state has a maximum number of International Airports?
A. 
Madhya Pradesh
B. 
Karnataka
C. 
Tamil nadu 
D. 
Kerala

Status :  

Correct
Answer : D
View Solution
Kerala has maximum 4 international Airports.

1. Kannur International Airport


2. Cochin International Airport
3. Kozhikode International Airport
4. Trivandrum International Airport

Bookmark Share question feedback

Type: MCQ Single    |    Marks: +1/-0   |    Topic: General Awareness > Static    |  


Time Spent: 
0 Mins 11 Secs
24. 
The Supreme Court can issue writs under which Article(s) of the constitution?
A. 
Article 32
B. 
Article 139
C. 
Article 226
D. 
Both 1 and 2

Status :  

Correct
Answer : D
View Solution
The Supreme Court can issue writs under Articles 32 and 139 of the constitution.
The type of writs include Habeas Corpus, Mandamus, Prohibition, Certiorari and Quo
warranto.

Article 32 Enforcements of Fundamental Rights

Article 139 Enforcements of rights excluding Fundamental Rights

Article 226 Gives power to High Courts to issue writs

 NOTE : Article 226 gives power to high courts to issue writs, not to Supreme Court.
Bookmark Share question feedback

Type: MCQ Single    |    Marks: +1/-0   |    Topic: General Awareness > Static    |  


Time Spent: 
0 Mins 21 Secs
25. 
Who among the following has won the Mohammed Rafi Award 2018?
A. 
Laxmikant Shantaram Kudalkar
B. 
Sanjeev Kohli
C. 
Madan Mohan
D. 
Yash Raj

Status :  

Incorrect
Given Answer : C
Answer : A
View Solution

 The 'Mohammed Rafi Award 2018' was given to late music composer Laxmikant
Shantaram Kudalkar and playback singer Usha Timothy.
  Laxmikant was given the Mohammed Rafi Lifetime Achievement Award
consisting of 1 lakh rupees and a trophy.
 Usha Timothy was honoured with the award and 51000 rupees.
 The awards were instituted by the NGO Spandan Arts. 

Bookmark Share question feedback

Type: MCQ Single    |    Marks: +1/-0   |    Topic: General Awareness > Static    |  


Time Spent: 
0 Mins 15 Secs
26. 
What is the official language of Daman & Diu?
A. 
Gujarati
B. 
English
C. 
Hindi
D. 
Portrugese 

Status :  

Correct
Answer : A
View Solution
The officail language of Daman & Diu is Gujarati. Gujarati is the predominant language
of Daman & Diu and is the most widely spoken.
Bookmark Share question feedback

Type: MCQ Single    |    Marks: +1/-0   |    Topic: General Awareness > Static    |  


Time Spent: 
0 Mins 21 Secs
27. 
Who invented the revolver?
A. 
Edward Jenner
B. 
Alexander Fleming
C. 
Albert Einstein
D. 
Samuel Colt

Status :  

Correct
Answer : D
View Solution
Samuel Colt was an American Industrialist and businessman who is credited with the
invention of Revolver.
Bookmark Share question feedback

Type: MCQ Single    |    Marks: +1/-0   |    Topic: General Awareness > Static    |  


Time Spent: 
0 Mins 15 Secs
28. 
Which of the following cities is known as "City of Lakes, " the Venice of the East, " or the
"Kashmir of Rajasthan? 
A. 
Jaipur
B. 
Udaipur
C. 
Kota
D. 
Jodhpur

Status :  

Correct
Answer : B
View Solution
Udaipur is the city which is called the "City of Lakes, " the Venice of the East, " or the
"Kashmir of Rajasthan. This is because of the many lakes are there in the city which add
to the beauty of Udaipur.
Bookmark Share question feedback

Type: MCQ Single    |    Marks: +1/-0   |    Topic: General Awareness > Current Affairs  


|    Time Spent: 
0 Mins 12 Secs
29. 
What is the theme of the National Girl Child Day (NGCD-2019)?
A. 
 
Beti Bachao, Beti Padhao 
B. 
Protect, Educate and Value Girls
C. 
Protect Future, Protect Girl
D. 
Empowering Girls for a Brighter Tomorrow

Status :  

Incorrect
Given Answer : A
Answer : D
View Solution
The National Girl Child Day (NGCD) is celebrated every year on January 24 in India to
raise awareness against the social stigma and discrimination faced by girl. The day also
highlights the need to protect girl child who is prone to sex selective abortions, sexual
abuse and trafficking in India. The 2019 theme “Empowering Girls for a Brighter
Tomorrow” was celebrated with objectives of generating awareness on the issue of
declining Child Sex Ratio (CSR) and create a positive environment around valuing the
girl child.
Bookmark Share question feedback

Type: MCQ Single    |    Marks: +1/-0   |    Topic: General Awareness > Current Affairs  


|    Time Spent: 
0 Mins 12 Secs
30. 
Which two countries officially quit the United Nations Educational, Scientific and Cultural
Organization (UNESCO)?
A. 
Italy and Libya 
B. 
Argentina And Algeria
C. 
Brazil And Cuba
D. 
United States And Israel

Status :  

Incorrect
Given Answer : C
Answer : D
View Solution
The United States and Israel officially quit the United Nations Educational, Scientific and
Cultural Organization (UNESCO) with the close of 2018. The countries had announced
their decisions in 2017 of withdrawing from the agency, accusing it of bias against Israel.
Bookmark Share question feedback

Type: MCQ Single    |    Marks: +1/-0   |    Topic: General Awareness > Current Affairs  


|    Time Spent: 
0 Mins 22 Secs
31. 
Who became the first Hindu woman to have been appointed as a civil judge in Pakistan?
A. 
Suman Kumari
B. 
Aashi Devi
C. 
Sunita Singh
D. 
Barkha Gupta

Status :  

Unanswered
Answer : A
View Solution
Suman Kumari has become the first Hindu woman to have been appointed as a civil
judge in Pakistan after passing an examination for induction of judicial officers.
 
Bookmark Share question feedback

Type: MCQ Single    |    Marks: +1/-0   |    Topic: General Awareness > Current Affairs  


|    Time Spent: 
0 Mins 10 Secs
32. 
In which city of India IAF carried 'Vayu Shakti' exercise?
A. 
Pokhran
B. 
Pathankot
C. 
Ghaziabad
D. 
Gwalior

Status :  

Correct
Answer : A
View Solution
IAF conducted mega exercise 'Vayu Shakti' in Pokhran, Rajasthan. The exercise was
carried out day and night where the IAF showcased firepower capability of indigenously
developed platforms and efficacy of missiles.
Bookmark Share question feedback

Type: MCQ Single    |    Marks: +1/-0   |    Topic: General Awareness > Current Affairs  


|    Time Spent: 
0 Mins 11 Secs
33. 
Which Indian state has topped in budgetary practices, as per latest survey by
Transparency International (TI)?
A. 
Andhra Pradesh
B. 
Odisha
C. 
Assam
D. 
Punjab

Status :  

Incorrect
Given Answer : B
Answer : C
View Solution
As per latest survey conducted by Transparency International (TI), Assam has occupied
the top slot in the ranking of best practices followed by states in Budget formulation and
is followed by Andhra Pradesh and Odisha. The states which figured lower in the ranking
were Meghalaya, Manipur and Punjab.
Bookmark Share question feedback

Type: MCQ Single    |    Marks: +1/-0   |    Topic: General Awareness > Current Affairs  


|    Time Spent: 
0 Mins 14 Secs
34. 
Which of the following states has recently declared Indus river dolphin as the state's
aquatic animal?
A. 
Punjab
B. 
Jammu and Kashmir
C. 
Haryana
D. 
Himachal Pradesh

Status :  

Incorrect
Given Answer : D
Answer : A
View Solution
Punjab has recently declared the endangered Indus river dolphin as the state's aquatic
animal. The decision was recently taken at a meeting of the state board for wildlife
chaired by chief minister Captain Amarinder Singh. Indus Dolphin is an endangered
freshwater aquatic mammal found only in Indian and Pakistan in the Beas river.
Bookmark Share question feedback

Type: MCQ Single    |    Marks: +1/-0   |    Topic: General Awareness > Current Affairs  


|    Time Spent: 
0 Mins 6 Secs
35. 
Which Indian squash player has won the 2019 Seattle Open title?
A. 
Cyrus Poncha
B. 
Harinder Pal Sandhu
C. 
Ramit Tandon
D. 
Saurav Ghosal

Status :  

Unanswered
Answer : C
View Solution
Indian squash player Ramit Tandon has won the 2019 Seattle Open title by defeating
Mohamed El Sherbini of Egypt in the final in USA. It is his fourth title in his career. The
Seattle Open is a Professional Squash Association (PSA) challenger squash Tour event.
Bookmark Share question feedback

Type: MCQ Single    |    Marks: +1/-0   |    Topic: General Awareness > Current Affairs  


|    Time Spent: 
0 Mins 12 Secs
36. 
Which country's President has been elected as the chairman of the African Union (AU)?
A. 
Ethiopia
B. 
South Africa
C. 
Egypt
D. 
Zambia

Status :  

Incorrect
Given Answer : A
Answer : C
View Solution
Egyptian President, Abdel-Fattah el-Sissi has been elected as the chairman of the
African Union (AU) at the continental body's summit in Ethiopia. Egyptian President,
Abdel-Fattah el-Sissi has been elected as the chairman of the African Union (AU) at the
continental body's summit in Ethiopia.
Bookmark Share question feedback

Type: MCQ Single    |    Marks: +1/-0   |    Topic: General Awareness > Current Affairs  


|    Time Spent: 
0 Mins 39 Secs
37. 
Which international organisation has launched 'The State of the Global Climate' report? 
A. 
WMO
B. 
IPCC
C. 
IUCN
D. 
UNEP

Status :  

Correct
Answer : A
View Solution
The State of the Global Climate' report has been recently launched by the World
Meteorological Organization (WMO)
Bookmark Share question feedback

Type: MCQ Single    |    Marks: +1/-0   |    Topic: General Awareness > Current Affairs  


|    Time Spent: 
0 Mins 22 Secs
38. 
ISRO has successfully launched EMISAT and 28 customer satellites by which launch
vehicle?
A. 
PSLV-C44
B. 
PSLV-C46
C. 
PSLV-C47
D. 
PSLV-C45

Status :  

Incorrect
Given Answer : C
Answer : D
View Solution
On April 1, ISRO has successfully launched India’s latest EMISAT defence satellite from
the Sriharikota spaceport. In today’s mission, the rocket PSLV-C45 flew in a new
configuration called “QL”.
Bookmark Share question feedback

Type: MCQ Single    |    Marks: +1/-0   |    Topic: General Awareness > Current Affairs  


|    Time Spent: 
0 Mins 9 Secs
39. 
Who has won the 2019 Formula 1 Bahrain Grand Prix tournament?
A. 
Valtteri Bottas
B. 
Sebastian Vettel
C. 
Charles Leclerc
D. 
Lewis Hamilton

Status :  

Correct
Answer : D
View Solution
Lewis Hamilton, a British racing driver, has won the 2019 Formula 1 Bahrain Grand Prix
tournament after an engine problem hit runaway leader Charles Leclerc's Ferrari late in
the race.
Bookmark Share question feedback

Type: MCQ Single    |    Marks: +1/-0   |    Topic: General Awareness > Current Affairs  


|    Time Spent: 
0 Mins 16 Secs
40. 
India has recently evacuated peacekeeping CRPF contingent from which of the following
African countries?
A. 
Tunisia
B. 
Libya
C. 
Kenya
D. 
Ethiopia
Status :  

Incorrect
Given Answer : C
Answer : B
View Solution
India has recently evacuated its entire contingent of peacekeeping forces comprising 15
CRPF personnel from Tripoli after the situation suddenly worsened in Libya.
Bookmark Share question feedback

You might also like